ravieshwar.singh
Thanks Received: 0
Vinny Gambini
Vinny Gambini
 
Posts: 1
Joined: July 31st, 2011
 
 
 

Q17 - Legislator: The recently released crime

by ravieshwar.singh Fri Sep 02, 2011 9:56 pm

Q17.

This one I thought had to be B - I've seen this type of question before, in which someone claims that something caused something else when there could be some broader alternative explanation that caused both things. In this case, the legislator claims that areas in which the new law is being implemented has reduced the crime rate. The analyst, however, says that in areas where the law is not being implemented, crime has dropped in the same way.

Answer B seems to work because they could both be the effect of a better economy, or some other factor. If it is the case that the reduction of the crime rate is due to another factor, then the law hasn't reduced the crime rate, thus weakening the argument.

I don't understand how it's E - why does it matter that the legislator hasn't considered the effect in the absence of the cause? The legislator is simply arguing that the law has reduced crime in an area, irrespective of how the absence of the law affects crime in other areas.

Thanks!

-Ravi
 
chike_eze
Thanks Received: 94
Atticus Finch
Atticus Finch
 
Posts: 279
Joined: January 22nd, 2011
 
This post thanked 2 times.
 
trophy
Most Thanked
 

Re: Q17 - The recently released crime

by chike_eze Sat Sep 03, 2011 1:33 am

The analyst responds by undermining the Legislator's cause-and-effect argument. Many areas "lack legislation" yet reported "drop in crime rate". He weakens the cause -> effect by showing that in some cases the effect occurred without the cause.

i.e.,

Legislator: new law -> reduced crime rate
Analyst: reduced crime rate without new law

ravieshwar.singh Wrote:Q17.
Answer B seems to work because they could both be the effect of a better economy, or some other factor. If it is the case that the reduction of the crime rate is due to another factor, then the law hasn't reduced the crime rate, thus weakening the argument.

This is a legitimate method of undermining a cause-n-effect argument, but it wasn't used by the Analyst.

ravieshwar.singh Wrote:I don't understand how it's E - why does it matter that the legislator hasn't considered the effect in the absence of the cause? The legislator is simply arguing that the law has reduced crime in an area, irrespective of how the absence of the law affects crime in other areas.

E is correct!
Why? The Analyst attacks the Legislator's argument by showing effect without cause. The Legislator overlooked this possibility.

Remember, to weaken X caused Y (X -> Y)

We show either one of the following:

Y -> X
Z -> X and Y
X without Y (cause without effect)
Y without X (effect without cause)
User avatar
 
demetri.blaisdell
Thanks Received: 161
LSAT Geek
 
Posts: 198
Joined: January 26th, 2011
 
This post thanked 1 time.
 
 

Re: Q17 - The recently released crime statistics clearly show...

by demetri.blaisdell Sun Sep 04, 2011 4:59 pm

I think chike_eze did a great job of explaining why (E) is the correct answer choice. But what about (B)? (B) is one of those frustrating answer choices with general language. We have to carefully relate it back to the stimulus.

It's true that the analyst thinks the legislator has unreasonably concluded that one event (the legislation) has caused another event (the reduction in crime). But does the analyst argue that there is a possibility that something else (you said economy) caused both events? No! The two events mentioned by the legislator are the legislation and the crime drop not the two areas with a decrease in crime rate.

If (B) were the correct answer, the analyst would be saying that it's possible that another factor caused the legislation and the crime drop. Instead, the analyst is pointing to another place without the "alleged cause" that exhibits the same effect. You get that in (E).

(B) is a great answer choice because it contains a very subtle shift that's easy to miss. Let me know if that helps you. If you still have questions, ask away!

Demetri
 
charmayne.palomba
Thanks Received: 24
LSAT Geek
 
Posts: 18
Joined: July 06th, 2010
 
This post thanked 3 times.
 
 

Re: Q17 - The recently released crime statistics clearly show...

by charmayne.palomba Tue Nov 01, 2011 3:44 pm

PT46, S2, Q17 (Analyze Argument (procedure))

(E) is correct.


Our task in this question is to identify the argument strategy used by the analyst. We want to make sure we have a clear understanding of the argument structure before approaching the answer choices. In this case, the legislator is arguing that since statistics show that crime has dropped by one fourth over the four years since stiffer punishments were introduced, those new laws have reduced the crime rate. Pretty simple.

The analyst, however, isn’t convinced that the new laws caused the drop in crime rate. (As LSAT debaters, we should be suspicious of claims of causation, too!) The analyst backs up his skepticism by saying that the crime rate also drops in comparable areas that lack stiffer punishment legislation.

(A) The analyst isn’t questioning whether the statistics are accurate; he’s questioning the cause of the drop in crime rate shown by the statistics.

(B) Don’t be fooled by abstract language! Is the analyst saying that the legislator has failed to consider the possibility that both lowered crime rates and the new legislation are both the effects of some other, common cause? No! The possibility of a third factor causing both is a common means of questioning a claim of causation, and while the analyst is questioning the legislator’s causal claim (that the legislation caused the drop in crime) but there is no mention of a third factor causing both.

(C) The analyst simply doesn’t do this, so we can eliminate it quickly.

(D) The analyst makes no mention of the legislator’s character or motives.

We’re down to (E), which is correct. This is another abstract language answer choice, like (B), so approach it in the same manner. Ask yourself, is the analyst claiming that the legislator failed to consider how often a drop in crime rate has occurred in the absence of stiffer punishment legislation? Yes, that’s exactly what he’s doing by pointing to instances of lower crime rates in comparable areas that lack the legislation. This tactic effectively undermines the legislator’s claim that the law caused the drop in crime.
 
Yit HanS103
Thanks Received: 2
Vinny Gambini
Vinny Gambini
 
Posts: 24
Joined: November 07th, 2017
 
 
 

Re: Q17 - The recently released crime statistics clearly show...

by Yit HanS103 Sat Feb 03, 2018 12:35 am

charmayne.palomba Wrote:PT46, S2, Q17 (Analyze Argument (procedure))

(E) is correct.


Our task in this question is to identify the argument strategy used by the analyst. We want to make sure we have a clear understanding of the argument structure before approaching the answer choices. In this case, the legislator is arguing that since statistics show that crime has dropped by one fourth over the four years since stiffer punishments were introduced, those new laws have reduced the crime rate. Pretty simple.

The analyst, however, isn’t convinced that the new laws caused the drop in crime rate. (As LSAT debaters, we should be suspicious of claims of causation, too!) The analyst backs up his skepticism by saying that the crime rate also drops in comparable areas that lack stiffer punishment legislation.

(A) The analyst isn’t questioning whether the statistics are accurate; he’s questioning the cause of the drop in crime rate shown by the statistics.

(B) Don’t be fooled by abstract language! Is the analyst saying that the legislator has failed to consider the possibility that both lowered crime rates and the new legislation are both the effects of some other, common cause? No! The possibility of a third factor causing both is a common means of questioning a claim of causation, and while the analyst is questioning the legislator’s causal claim (that the legislation caused the drop in crime) but there is no mention of a third factor causing both.

(C) The analyst simply doesn’t do this, so we can eliminate it quickly.

(D) The analyst makes no mention of the legislator’s character or motives.

We’re down to (E), which is correct. This is another abstract language answer choice, like (B), so approach it in the same manner. Ask yourself, is the analyst claiming that the legislator failed to consider how often a drop in crime rate has occurred in the absence of stiffer punishment legislation? Yes, that’s exactly what he’s doing by pointing to instances of lower crime rates in comparable areas that lack the legislation. This tactic effectively undermines the legislator’s claim that the law caused the drop in crime.


I thought this was a "weaken question" even the answer choices are written as the ones found in "weaken questions"
I do not see this as "analyze the argument qs.- procedure". To confirm, I went back to the book (manhattan LR chapter 10 "analyze the argument questions- procedure" ) and the stem, plus answer choices are completely different than the ones used in weaken questions.
procedure qs., to my understanding ask more about the strategy used in the stimulus, right? Please, help, I'm confused.
thank you!